Storing Cookies (See : http://ec.europa.eu/ipg/basics/legal/cookies/index_en.htm ) help us to bring you our services at overunity.com . If you use this website and our services you declare yourself okay with using cookies .More Infos here:
https://overunity.com/5553/privacy-policy/
If you do not agree with storing cookies, please LEAVE this website now. From the 25th of May 2018, every existing user has to accept the GDPR agreement at first login. If a user is unwilling to accept the GDPR, he should email us and request to erase his account. Many thanks for your understanding

User Menu

Custom Search

Author Topic: Sum of torque  (Read 173222 times)

EOW

  • Sr. Member
  • ****
  • Posts: 383
Re: Sum of torque
« Reply #210 on: September 05, 2015, 10:27:06 PM »
With a unique disk +arm there is a problem with the linear momentum too. If I explode the disk after it rotates around itself the linear momentum is not conserved => antigravity device

EOW

  • Sr. Member
  • ****
  • Posts: 383
Re: Sum of torque
« Reply #211 on: September 10, 2015, 06:53:54 PM »
The spring like that.

The spring increases its length.

The red arm receives a positive torque.

The disk receives a positives torque.

EOW

  • Sr. Member
  • ****
  • Posts: 383
Re: Sum of torque
« Reply #212 on: September 11, 2015, 05:48:56 PM »
First idea:

I come back with the container full with springs and small balls. The pressure like a fluid inside the container. The part of the red solid is attrack with spring like that there is no force at "bottom". I need to take a special shape of the red solid, it is not a disk, it is an addition of circles in translation. With a circle I don't have a torque on the red solid from the pressure. With the shape I drawn I don't have a torque on the arm. The disk is always in the position I drawn because I accelerate the arm more and more.

The problem is I need to have an infinity of arms !

Second idea: start the devices each disk will rotates at w*cos(a) (not the perpendicular disk):

I can take 3 disks, 2disks and one disk in a perpendicular plane. I start the device, the 2 disks turns slower than the support so it's possible with friction between disks to accelerate the perpendicular disk, and in the same time 2 disks accelerates in the laboratory reference.
« Last Edit: September 11, 2015, 10:47:23 PM by EOW »

EOW

  • Sr. Member
  • ****
  • Posts: 383
Re: Sum of torque
« Reply #213 on: September 12, 2015, 10:17:48 AM »
With 2 plates free in space to move up or down. The plates are in rotation. There is friction between the plates and the orange disk.

Or with 2 grey circles fixed to the ground. The orange torus increases its angular velocity in the labo reference. Don't forget friction gives an energy of heating. the best is to have the friction in one direction higher than in other direction. I want to decelerate the torus in the arm reference with friction but in this case the friction will slow down the red arm. If I can have the friction of rotation around the magenta axis equal to zero it's better. I would like to have the forces F1 and F2 on the torus and no force from friction on the axis of the torus like that the red arm don't decelerate. Even the red arm decelerate due to friction the energy is not lost it is in heating.

EOW

  • Sr. Member
  • ****
  • Posts: 383
Re: Sum of torque
« Reply #214 on: September 12, 2015, 07:08:59 PM »
With the device in the same plane like I studied before. It's possible to use a belt. A unique belt. A belt without a mass. The belt has a friction. I use the friction of the belt to decrease the angular velocity of the disk (arm reference) like that I recover the energy from friction and the disk turns more in the garage reference.

I can take my idea of the message #74

http://overunity.com/15011/sum-of-torque/msg428104/#msg428104

The image case1.png

The belt is like a "8" shape.

The friction of the belt is not between the belt and the support nor the belt and the disk but when the belt changes its shape, at each time the belt change its angle.




EOW

  • Sr. Member
  • ****
  • Posts: 383
Re: Sum of torque
« Reply #215 on: September 12, 2015, 09:37:15 PM »
No gravity here.

I think like that it's a good device:

The torus has a mass.

1/ Turn the red arm at w. Turn the grey circles at 0.999w. Grey circles have mass. The angular velocity of the orange torus is -w*cos(a) in the arm reference with a the angle of the axis from the vertical.
2/ I use the torus like a catapult. The grey circles can be cut at small parts when I want. Like that the torus can have only a torque on it, the red arm don't receive a torque. The small parts of the circles can be eject and the velocity is higher than before the friction with the torus. Need to recover the energy from the small parts of circles. Small parts are free to move in 3d space. A small part has it velocity from the rotation and the velocity up/down from the torus, it's an addition of velocities (vectors).

It's not really a friction between the torus and the circle it's more use the torus like a catapult. The goal is to keep no torque on the grey circles.
« Last Edit: September 13, 2015, 08:42:56 AM by EOW »

EOW

  • Sr. Member
  • ****
  • Posts: 383
Re: Sum of torque
« Reply #216 on: September 13, 2015, 11:31:48 AM »
I add an image in 3d to look the orange disk between the grey torus.

The energy won is : 2*(-1/2*I*(w*(1-cos(a)))^2) + (1/2*I*w^2)) = 2*1/2*I*(-(-2w^2cos(a)+w^2*cos²(a))) = Iw² (2cos(a)-cos²(a))
I: the inertia of the orange torus around itself.
a: the angle of the axis of the orange torus from the vertical.

In practice the vectors of velocities are not perfectly perpendicular (i8.png), V2 cancel a little of V1 but V3 add a little of V1, the sum is near 0.

EOW

  • Sr. Member
  • ****
  • Posts: 383
Re: Sum of torque
« Reply #217 on: September 14, 2015, 12:29:35 PM »
With the device of the video :

https://www.youtube.com/watch?v=Pjc4dIf1aWI&feature=youtu.be

I use 2 devices like this I turn one red arm clockwise and the another red arm counterclockwise and I exchange the torus ! I lost an energy.

Or I use friction between the disks
« Last Edit: September 14, 2015, 08:45:06 PM by EOW »

EOW

  • Sr. Member
  • ****
  • Posts: 383
Re: Sum of torque
« Reply #218 on: September 15, 2015, 12:36:32 PM »
Cycle:

1/ I launch the red arm and the green arm at the angular velocity w. The orange disk turn around itself at w*(1-cos(a)) in the laboratory reference. The blue disk don't turn around itself.
2/ I use stems (like a parts of belt) for give the forces F3 and F4 to the blue disk. The blue disk increases its angular velocity around itself. The orange disk increases its angular velocity around itself too. The red arm and the green arm don't receive a torque.

« Last Edit: September 15, 2015, 05:47:22 PM by EOW »

EOW

  • Sr. Member
  • ****
  • Posts: 383
Re: Sum of torque
« Reply #219 on: September 16, 2015, 09:11:56 AM »
And sure, I can do this with spring(s) no need the blue disk. Replace the stem by spring(s). There is a positive torque on the red arm. The orange disk increases its angular velocity around itself. The device must turn at an infinite angular velocity. The distance of the spring never change because the arm accelerates more and more. The spring is always like I drawn: vertical.
« Last Edit: September 16, 2015, 06:52:57 PM by EOW »

EOW

  • Sr. Member
  • ****
  • Posts: 383
Re: Sum of torque
« Reply #220 on: September 16, 2015, 09:59:22 PM »
Last is bad, the disk turn in the opposite direction so I can do this :


EOW

  • Sr. Member
  • ****
  • Posts: 383
Re: Sum of torque
« Reply #221 on: September 17, 2015, 11:19:45 PM »
With a thickness for the disks I can have a difference of local velocity to have the forces F1 and F2. No torque on arms. Like each disk don't turn around itself (without friction), F1 and F2 will change the angular velocity of each disk so the kinetic energy increases.

EOW

  • Sr. Member
  • ****
  • Posts: 383
Re: Sum of torque
« Reply #222 on: September 18, 2015, 04:04:16 PM »
Like that the cylinder receive a positive torque like the arm from F1. The torque from F2 is cancelled by F3y. F3x don't give a torque. I want to keep constant the length of each spring, for that I need to accelerate the arms like the cylinder accelerate. Even the torque from F1 and F2 are not enough I will accelerate the arms like the cylinder. I recover the energy later.
« Last Edit: September 18, 2015, 06:55:29 PM by EOW »

EOW

  • Sr. Member
  • ****
  • Posts: 383
Re: Sum of torque
« Reply #223 on: September 19, 2015, 06:30:32 AM »
I added some informations on the image (I didn't draw all springs, but there is one for one ball and some for the sphere), the balls in reality must be smaller than I drawn, it's a theoretical study where I suppose I can obtain the pressure of a fluid under gravity inside the container. The sphere is a full object, there isn't balls between the sphere and the container but I attract the sphere with the springs.

1/ The sphere receives a clockwise torque from the springs
2/ The part of the sphere in contact with balls don't receive a torque because the shape is an addition of part of circles
3/ The red arm receives a force F2 because the springs attract the spehre but balls give a force F3 (composed of F3x, F3y and F3z), and F3y=-F2 (in vectors) so there is no torque from the sum F2 and F3y
4/ F3z and F3x can't give a torque on the red arm
5/ There is a surface with balls at left greater than at right in the container because at right there is the sphere . Like the pressure is higher at "bottom" the force by unit of surface is greater at "bottom" so there is a clockwise torque on the black arm, I resumed this torque by the force F1
6/ At "bottom" there is no force because springs attract all balls and the volume of the sphere

There is a last problem, the sphere will receive a torque on it and like it must turn at the angular velocity w too it seems the sphere lost the energy I won with the black arm. Now, imagine all the mass of the sphere is in the center (like particule in physics, you know, some people say: it's a point particle with mass), so if the mass is in the center I don't lost any energy from the sphere because the energy of rotation is 1/2Iw² and I is at 0 with a point particle. I need the sphere with a radius but I want the mass in the center. In our reality, the mass can't be at a point but can be near the center, so the energy lost by the sphere can be small.


The black arm will receive a torque. The red arm don't receive any torque. Two arms turn at w. There is a last problem, the sphere will receive a torque on it and like it must turn at the angular velocity w too it seems the sphere lost the energy I won with the black arm. Now, imagine all the mass of the sphere is in the center (like particule in physics, you know, some people say: it's a point particle with mass maybe it's not really a point), so if the mass is in the center (or near the center) I don't lost any energy from the sphere because the energy of rotation is 1/2Iw² and I is at 0 with a point particle. I need the sphere with a radius but I want the mass in the center. In our reality, the mass can't be at a point but can be near the center, so the energy lost by the sphere can be small.

Maybe like this the inertia is not very high for you. So, in this case, with the mass in the wall of the sphere (or in the center) when I apply a torque on it, I apply a torque on a gyroscope if 'w' is high so the sphere turns in the perpendicular axis like a gyroscope can do and don't lost its energy.

If you don't want a high 'w' you place the mass in the center of the sphere and turn it very quickly around this axis you want like that when the springs want to turn the sphere it turns a gyroscope. Sure I use springs for show the device but imagine an electrostatic force for example.

And to be ok with my theory with gravitation, I need 'w' very high, it's great because the precession of the gyroscope is a function of 'w', higher is 'w' lower is the precession.
« Last Edit: September 19, 2015, 06:12:58 PM by EOW »

EOW

  • Sr. Member
  • ****
  • Posts: 383
Re: Sum of torque
« Reply #224 on: September 19, 2015, 06:45:05 PM »
The sphere will works like a gyroscope. Put the mass in the center and turn it around itself at a very high angular velocity w'. The black arm receives a torque and can increase the energy of the device. The sphere receive a torque on it too, this could cancel the energy won by the arm but if the sphere is like a gyroscope, the gyroscope precession.

The precession of the gyroscope is very low (low inertia but a very high angular velocity) and in one turn the mass return to its initial position because the recipient keeps it position.
« Last Edit: September 19, 2015, 11:05:01 PM by EOW »